LSAT and Law School Admissions Forum

Get expert LSAT preparation and law school admissions advice from PowerScore Test Preparation.

User avatar
 Dave Killoran
PowerScore Staff
  • PowerScore Staff
  • Posts: 5925
  • Joined: Mar 25, 2011
|
#88033
Complete Question Explanation
(The complete setup for this game can be found here: lsat/viewtopic.php?f=164&p=88024#p88024)

The correct answer choice is (A).

In a Global Could Be True question, the usual plan of attack should be to use the rules to eliminate incorrect answer choices, and then to use the inferences.

In this question, the first rule eliminates three answer choices:

Answer choice (B) can be eliminated because if G and V are on the same team, then M cannot be paired with either.

Answer choice (D) can be eliminated because M must be teamed with G or V.

Answer choice (E) can be eliminated because M must be teamed with G or V.

The second rule can be used to eliminate answer choice (C). Thus, answer choice (A) is correct.

Get the most out of your LSAT Prep Plus subscription.

Analyze and track your performance with our Testing and Analytics Package.